Fourth Order Homogenous Differential Equation

Click For Summary
The discussion centers on solving a fourth-order homogeneous differential equation with given initial conditions. The characteristic equation yields roots of 0 and 3, with 0 being a repeated root. The general solution is expressed as a linear combination of the basic set of solutions: {1, x, e^(3x), xe^(3x)}. The initial conditions are then used to determine the constants in the general solution. Clarification on the form of the solution and the handling of repeated roots helps resolve confusion regarding the approach.
shards5
Messages
37
Reaction score
0

Homework Statement


y4 - 6y3 + 9y2
y(0) = 19; y'(0) = 16; y"(0) = 9; y"'(0) = 0


Homework Equations


N/A

The Attempt at a Solution


Factored out the equation and obtained the following roots.
r2(r2-3) = 0 which gives r = 0 and r =3.
Using those roots, I make the following general solution.
y(x) = j*e3x + k*x*e3x + L*k*x2*e3x
I am assuming since one of the roots is zero then the solution will not have to have add i*e0t. I am also assuming since this is a fourth order equation that I will need to solve for n=4 variables and that this is the form in which I should tackle it. Am I mistaken in my assumptions?
 
Physics news on Phys.org
Is your differential equation? y(4) - 6y(3) + 9y(2) = 0?

Your characteristic equation has four roots, with 0 and 3 repeated. Your basic set of solutions is {1, x, e3x, xe3x}. Your general solution will be all linear combinations of these functions, or
y(x) = c1*1 + c2*x + c3*e3x + c4*xe3x. Use your initial conditions to solve for the constants ci.
 
Yea it was equal to zero and thanks for the general equation. It solved a lot of the confusion I had on what to do with the r = 0.
 
Question: A clock's minute hand has length 4 and its hour hand has length 3. What is the distance between the tips at the moment when it is increasing most rapidly?(Putnam Exam Question) Answer: Making assumption that both the hands moves at constant angular velocities, the answer is ## \sqrt{7} .## But don't you think this assumption is somewhat doubtful and wrong?

Similar threads

Replies
7
Views
2K
  • · Replies 27 ·
Replies
27
Views
2K
Replies
1
Views
1K
  • · Replies 7 ·
Replies
7
Views
2K
  • · Replies 2 ·
Replies
2
Views
1K
Replies
2
Views
1K
  • · Replies 4 ·
Replies
4
Views
2K
Replies
7
Views
2K
  • · Replies 10 ·
Replies
10
Views
2K